LSAT and Law School Admissions Forum

Get expert LSAT preparation and law school admissions advice from PowerScore Test Preparation.

User avatar
 Dave Killoran
PowerScore Staff
  • PowerScore Staff
  • Posts: 5850
  • Joined: Mar 25, 2011
|
#27062
Complete Question Explanation
(The complete setup for this game can be found here: lsat/viewtopic.php?t=8627)

The correct answer choice is (C)


When R is reduced, N cannot be reduced. When both M and R are reduced, L cannot be reduced. Answer choice (C), which contains both L and N, is therefore correct.
User avatar
 cpihl13
  • Posts: 7
  • Joined: Mar 12, 2021
|
#85424
For Question #11 I see how answer choice C can work, but couldn't answer A work as well? I was stuck between those two and it seemed like either would work. If M and R are reduced, then L is not. Therefore, wouldn't P have to be reduced because L is reduced, since both can't both be reduced or not reduced? Then, that would mean that S has to be reduced since it can not reduced like N? That would mean that G would have to not be reduced because if it was reduced then W would need to be reduced and there is no room.

Thanks for your help!
User avatar
 Dave Killoran
PowerScore Staff
  • PowerScore Staff
  • Posts: 5850
  • Joined: Mar 25, 2011
|
#85429
Hi C,

It's close, but you made a mistake in your reasoning. The good news is that now that you ran into this problem, you won't make the same mistake again (and it's a common one so it's good to learn from it now :-D )

As you worked this out, you said that, "Therefore, wouldn't P have to be reduced because L is reduced, since both can't both be reduced or not reduced?" This isn't the case, so let's cover why.

The rule states that, "If P is reduced, L is not reduced." This rule just eliminates one specific scenario: P and L both being reduced. Any other combination is still possible, including the following:

  • 1. P reduced, L not reduced.
    2. L reduced, P not reduced.
    3. Neither P nor L is reduced.
It's this third option you ruled out, but it is still possible.

So, in answer choice (A), what we could have is:

  • M and R are reduced, and automatically L cannot be reduced.
    Since R is reduced, N cannot be reduced. [which at this point proves (C)]
    Now, we have the choice of G, S, P, and W. So, we could have G, S, W if we wanted, and that rules out answer choice (A).
Please let me know if that helps. Thanks!
User avatar
 cpihl13
  • Posts: 7
  • Joined: Mar 12, 2021
|
#85456
Hi Dave,
Yeah I realized after posting that I made a mistake but you helped clarify it!

Thanks so much,
-Cole
 kliu49
  • Posts: 11
  • Joined: Jan 13, 2019
|
#90094
Hello all,

Please help me crack this. This question is labeled as a cannot be true, yet the question states "which one of the following is a pair of areas neither of which COULD be reduced?"

So for example, option B COULD be true. Here is a scenario: MRSPW reduced, LNG not reduced.

I understand LN is the only answer if the question stem asked "which one of the following is a pair of areas neither of which CANNOT be reduced".

What am I missing? Thank you in advance!
User avatar
 atierney
PowerScore Staff
  • PowerScore Staff
  • Posts: 215
  • Joined: Jul 06, 2021
|
#90117
So, we have Cinderella, and her secret sister (who didn't get a fairytale), Spinderella (maybe it's Sleeping Beauty?)... anyway, so the mother says... "Cinderella, Spinderella, neither of you two can go to the ball today!"

Now ask yourself how you would interpret this. "neither of the two" is the same as "neither of which," where "which" is not a wicked step mom!

The idea here is that neither of which means not any of the two CAN go to the ball. Here, the same reasoning would apply. Neither of the two in the correct answer choice can be reduced, so neither of the two can be selected. And answer choice C would be the correct answer.

The other way you suggested is a double negative, and not simply of the "Can't never could" variety! I won't not tell you that when you have negatives, you don't not cancel them out to form an affirmative! Neither Cinderella nor Spinderella didn't go to the ball, so guess what? The lucky prince was seeing double... and I'm not not sure it was not a negative!

Get the most out of your LSAT Prep Plus subscription.

Analyze and track your performance with our Testing and Analytics Package.